English Deutsch Français Italiano Español Português 繁體中文 Bahasa Indonesia Tiếng Việt ภาษาไทย
All categories

Could you please, instead of just giving the answer, take me step by step through the process?
Thanks! Your help is much appreciated ^^

2007-07-07 01:07:09 · 5 answers · asked by Anonymous in Science & Mathematics Mathematics

5 answers

Sorry, this statement is palpably false!
In fact, the only integer divisible by all positive integers
n is 0.
Proof:
1 is only divisible by 1.
Let k be any integer greater than 1.
Then we know
by the prime decomposition law,
k = p1^a1... pt^at,
where p1, ... , pt are distinct primes.
In particular, k is divisible by only a finite
number of primes.
But we know there are infinitely many primes,
so there are infinitely many primes not dividing k.
Now we have to prove that
43^(n+83) + 92^(3n-1) is not zero.
But that's clear, because both terms of
the sum are positive for all n.

2007-07-07 06:30:30 · answer #1 · answered by steiner1745 7 · 0 0

enable's use mod 7 arithmetic(called calendar arithmetic). (think of of seven days as a result as touchdown on the comparable day of the week.) i assume you're finding at 40 3 ^n + eighty 3(ninety two ^(3n-a million) ) here. actual, extra is actual: 40 3^m + eighty 3*(ninety two^n) is divisible via 7 for ALL valuable integers m and n. look: 40 3 = a million(mod 7) So 40 3^m = a million(mod 7) for all m. next, ninety two = a million(mod 7) and eighty 3 = -a million(mod 7) So 40 3^m + eighty 3*(ninety two^n) = a million - a million = 0(mod 7).

2016-12-10 04:41:18 · answer #2 · answered by walpole 4 · 0 0

{43^(n+83)} * {92^(3n-1)}

... will always work if n = 1.
... will still work if n = 2 (since 92 is even)
but 43^86 * 92^8 is not divisible by 3 since none of the original bases are divisible by 3 to start with.

The statement fails.
d:

2007-07-07 01:21:18 · answer #3 · answered by Alam Ko Iyan 7 · 2 1

This is time taking or I would do it for u..
sorryyyyyyyyyyyyyyyyyy/...............

2007-07-07 01:10:14 · answer #4 · answered by hindol g 1 · 0 1

No Thanks, I'm full.

2007-07-07 01:09:11 · answer #5 · answered by kevrigger 5 · 1 1

fedest.com, questions and answers